Menopause Flashcards

1
Q

A major histologic ovarian structural change associated with menopause is proliferation of:

A. theca interna cells.
B. granulosa cells.
C. epithelial surface cells.
D. stromal cells.

A

D. stromal cells.

O

Explained

https://digitum.um.es/digitum/bitstream/10201/29790/1/Human%20postmenopausal%20ovary%20-%20hormonally%20inactive%20fibrous%20connective%20tissue%20or%20more.pdf

How well did you know this?
1
Not at all
2
3
4
5
Perfectly
2
Q

Which one of the following would be MOST likely to contribute to osteoporosis?

A. Phenytoin
B. Amitryptiline
C. Captopril
D. Warfarin

A

A. Phenytoin

How well did you know this?
1
Not at all
2
3
4
5
Perfectly
3
Q

A 50 year old patient has received conflicting advice concerning hormone therapy (HT). Which of the following would be CORRECT information to relate to this patient?

A. Bone loss is most rapid in the first 3 years after the onset of menopause
B. Ingestion of 1,500 mg of elemental calcium per day will lead to an increased incidence of kidney stones
C. HT is not indicated after the age of 60
D. HT increases the incidence of coronary heart disease
E. HT decreases the risk of breast cancer

A

A. Bone loss is most rapid in the first 3 years after the onset of menopause

O

How well did you know this?
1
Not at all
2
3
4
5
Perfectly
4
Q

Which of the following statements about the recognised unwanted effects of drugs is correct?

a. Hirsutism is associated with cyproterone acetate.
b. Hot flushes are associated with clomiphene citrate.
c. Hypertension is associated with bromocriptine.
d. Fetal virilisation is associated with medroxyprogesterone acetate.

A

b. Hot flushes are associated with clomiphene citrate.

Orthostatic hypotension = side effect of bromocriptine

How well did you know this?
1
Not at all
2
3
4
5
Perfectly
5
Q

A 65-year-old woman has recovered from a Colles’ fracture. A recent bone scan reports decreased bone density. Her decreased bone mass is principally due to:
Select one:

a. increased osteoclastic activity.
b. decreased intestinal calcium absorption.
c. decreased osteoblastic activity.
d. decreased Vitamin D levels

A

a. increased osteoclastic activity.

How well did you know this?
1
Not at all
2
3
4
5
Perfectly
6
Q

A 53-year-old postmenopausal woman has been reading about Tibolone and would prefer to use it rather than oestrogen/progestogen therapy (Menopause Replacement Therapy-MRT).
You tell her that Tibolone:

a. is as effective as bioequivalent doses of combined MRT in reducing the frequency of vasomotor symptoms.
b. has a reduced incidence of vaginal bleeding compared with combined MRT.
c. decreases the risk for endometrial cancer compared with placebo.
d. has less risk for breast cancer recurrence than placebo.

A

b. has a reduced incidence of vaginal bleeding compared with combined MRT.

How well did you know this?
1
Not at all
2
3
4
5
Perfectly
7
Q

Which of the following is most correct concerning bone mass?

a. Bone mass peaks in most women before 25 years of age.
b. The accelerated loss in bone mass begins after menopause.
c. Bone mass is approximately 65% cortical and 35% trabecular bone.
d. A 70-year-old woman, not on hormone therapy, would lose approximately 2% of bone mass per year.

A

a. Bone mass peaks in most women before 25 years of age.

O

Approximately 80% of the bone mass is in the cortical compartment.
Bone mass peaks 20-30y

The annual rate of postmenopausal bone loss has been calculated to be 1.3–1.5 percent at the lumbar spine and 1.4 percent at the femoral neck

How well did you know this?
1
Not at all
2
3
4
5
Perfectly
8
Q

Subcut oestradiol therapy exhibits the following differences when compared with oral oestrogen

a) Physiological ratio of estradiol to estrone is achieved
b) Rarely causes endometrial hyperplasia
c) Leads to more abnormal changes in clotting factors
d) Is less likely to increase serum level of rennin substrate
e) More effective in prevention of osteoporosis

A

a) Physiological ratio of estradiol to estrone is achieved

As per RANZCOG MCQ

How well did you know this?
1
Not at all
2
3
4
5
Perfectly
9
Q

The MOST COMMON benign ovarian tumor found in postmenopausal women is

A. cystic teratoma
B. serous cystadenoma
C. mucinous tumor
D. endometrioid tumor

A

B. serous cystadenoma

O

How well did you know this?
1
Not at all
2
3
4
5
Perfectly
10
Q

Which one of the following is LEAST TRUE of osteoporosis?

A. Biphosphonates have compatible efficacy to oestrogen in both the prevention and treatment of osteoporosis
B. Hepatic impairment limits the use of biphosphonates but they are well tolerated orally
C. Calcitonin is effective therapy for osteoporosis but must be given nasally or subcutaneously as it is not effective orally
D. Tamoxifen increases bone mineral density when compared to placebo in postmenopausal women

A

B. Hepatic impairment limits the use of biphosphonates but they are well tolerated orally

*Minimal hepatic metabolism, avoid with significant renal impairment

How well did you know this?
1
Not at all
2
3
4
5
Perfectly
11
Q

Which of the following is NOT a risk factor for osteoporosis?

A. Caucasian race
B. Immobility
C. Chronic alcoholism
D. Obesity

A

D. Obesity

O

Increased peripheral oestrogen

How well did you know this?
1
Not at all
2
3
4
5
Perfectly
12
Q

Which of the following is LEAST CORRECT with respect to the WHI trial of HRT?

A. Fractures are decreased by HRT
B. Total cancer incidence is similar in HRT and no HRT groups
C. HRT increases the incidence of myocardial infarction and cerebrovascular accidents, even after adjusting for risk factors for arteriosclerosis
D. The trial included a substantial proportion of women with risk factors for arteriosclerosis, including HTN, age >70 and DM

A

C. HRT increases the incidence of myocardial infarction and cerebrovascular accidents, even after adjusting for risk factors for arteriosclerosis

O

How well did you know this?
1
Not at all
2
3
4
5
Perfectly
13
Q

A slim 58yo who experienced menopause 6 years ago and has taken intermittent oestrogen replacement until 6 months ago, consults you for HRT. She has debilitating hot flushes. She has a negative family history and doesn’t smoke. Physical and pelvic examination reveals a pale vagina, small uterus and cervix. You counsel her that the BEST hormone replacement is

a. Cyclic CEE 0.625mg days 1-25, and MPA 10mg days 16-25
b. Sequential CEE 0.625mg daily, and MPA 10mg days 21-30
c. CEE 0.625mg and MPA 5mg daily
d. CEE 0.625mg days 1-25, MPA 10mg days 13-25

A

c. CEE 0.625mg and MPA 5mg daily

O

Use continuous is >12 months amenorrhoea or >12 months of cyclical HRT done

How well did you know this?
1
Not at all
2
3
4
5
Perfectly
14
Q

Which of the following is LEAST correct concerning Tibolone?

A. It alleviates postmenopausal vasomotor symptoms
B. It does not stimulate the endometrium
C. It prevents osteoporosis
D. It inhibits proliferation of human breast cells
E. It improves the lipid profile with an increase in serum HDL cholesterol
F. It lowers sex hormone binding globulin increasing the bioavailability of endogenous testosterone

A

E. It improves the lipid profile with an increase in serum HDL cholesterol

O

Synthetic steroid

How well did you know this?
1
Not at all
2
3
4
5
Perfectly
15
Q

The benefits of oestrogen replacement therapy in elderly women are LEAST well proven for the prevention of:

A. osteoporosis
B. skin wrinkling
C. urge incontinence
D. recurrent UTIs

A

B. skin wrinkling

O

How well did you know this?
1
Not at all
2
3
4
5
Perfectly
16
Q

A 60yo postmenopausal woman, not on HRT presents with postmenopausal bleeding Which if the following is the MOST likely reason?

A. Genital tract malignancy
B. Endometrial polyps
C. Atrophic changes of the endometrium
D. Degeneration of uterine fibroids
E. Atrophic vaginitis
A

C. Atrophic changes of the endometrium

As per RANZCOG MCQs August 2008

17
Q

A 50yo well woman attends for regular check up. Hysterectomy for fibroids at 35yo. No symptoms. FSH 60. What regime of HRT would you prescribe?

a. continuous EE 30mcg and cyclic NE 10mg
b. continuous Premarin, cyclic Primolut
c. continuous low dose conjugated E, 2.5mg Provera daily
d. no clinical need for HRT

A

d. no clinical need for HRT

Milford

Is asymptomatic!

18
Q

A 53 yo multiparous postmenopausal woman has recently begun daily therapy with Premarin 1.25mg to prevent osteoporosis. She is non-obese but hypertensive and has an intact uterus. The patient is concerned about the risk of endometrial carcinoma with ERT and comes to you for a second opinion. You explain that her greatest risk of developing endometrial carcinoma relates to?

a. overabundance of endogenous oestrone
b. hypertension
c. development of and oestrogen secreting tumour
d. unopposed ERT

A

d. unopposed ERT

Milford

19
Q

50 yo whose last menstrual period was 6 months would like some advice.

a. Continue contraception for another 5 yrs
b. Warn of dangers of possible pregnancy
c. Wait for another 6 months before commencing HRT
d. Advise hysterectomy

A

b. Warn of dangers of possible pregnancy

Milford

20
Q

Vasomotor irritability found in postmenopausal women is due to?

a) Level of oestrogen
b) Alteration in thermal regulation
c) LH surge
d) Elevated FSH
e) Level of progesterone

A

b) Alteration in thermal regulation

M

21
Q

A 45year old Caucasian woman who has been smoking a pack of cigarettes a day for the past 20 years asks your advice about hormonal replacement therapy. Her LMP was 6months ago and she has been having occasional hot flushes. Which of the following statements about hormone therapy is ACCURATE?

A. Since LMP was only 6months ago she should reassess HRT use in another 6months.
B. HRT is contraindicated if she continues to smoke.
C. Addition of an progestogen for 7 days at the end of each month is adequate to protect against endometrial hyperplasia
D. HRT will reduce the increased risk of osteoporosis associated with smoking

A

D. HRT will reduce the increased risk of osteoporosis associated with smoking

O

22
Q

Which of the following is LEAST likely to help with hot flushes?

a. Venlafaxine
b. Fluoxetine
c. Clonidine
d. Phytoestrogens

A

d. Phytoestrogens

O

  • weak oestrogens found in plants
  • not shown to reduce VMS
23
Q

A 45year old Caucasian woman who has been smoking a pack of cigarettes a day for the past 20 years asks your advice about hormonal replacement therapy. Her LMP was 6months ago and she has been having occasional hot flushes. Which of the following statements about hormone therapy is ACCURATE?

A. Since LMP was only 6months ago she should reassess HRT use in another 6months.
B. HRT is contraindicated if she continues to smoke.
C. Addition of an progestogen for 7 days at the end of each month is adequate to protect against endometrial hyperplasia
D. HRT will reduce the increased risk of osteoporosis associated with smoking

A

D. HRT will reduce the increased risk of osteoporosis associated with smoking

O